Difference between revisions of "2021 Fall AMC 12B Problems/Problem 10"

(I should receive credit for the diagram, and we combine solution (coauthored).)
m (Video Solution by Beauty of Math)
Line 70: Line 70:
 
~Steven Chen (www.professorchenedu.com) ~Wilhelm Z ~MRENTHUSIASM
 
~Steven Chen (www.professorchenedu.com) ~Wilhelm Z ~MRENTHUSIASM
  
== Video Solution by Beauty of Math ==
+
== Video Solution by TheBeautyofMath ==
 
https://youtu.be/4qgYrCYG-qw?t=1304
 
https://youtu.be/4qgYrCYG-qw?t=1304
 +
 +
~IceMatrix
  
 
== See Also ==
 
== See Also ==
 
{{AMC12 box|year=2021 Fall|ab=B|num-a=11|num-b=9}}
 
{{AMC12 box|year=2021 Fall|ab=B|num-a=11|num-b=9}}
 
{{MAA Notice}}
 
{{MAA Notice}}

Revision as of 00:44, 30 December 2022

Problem

What is the sum of all possible values of $t$ between $0$ and $360$ such that the triangle in the coordinate plane whose vertices are \[(\cos 40^\circ,\sin 40^\circ), (\cos 60^\circ,\sin 60^\circ), \text{ and } (\cos t^\circ,\sin t^\circ)\] is isosceles?

$\textbf{(A)} \: 100 \qquad\textbf{(B)} \: 150 \qquad\textbf{(C)} \: 330 \qquad\textbf{(D)} \: 360 \qquad\textbf{(E)} \: 380$

Solution

Let $A = (\cos 40^{\circ}, \sin 40^{\circ}), B = (\cos 60^{\circ}, \sin 60^{\circ}),$ and $C = (\cos t^{\circ}, \sin t^{\circ}).$ We apply casework to the legs of isosceles $\triangle ABC:$

  1. $AB=AC$

    Note that $A$ must be the midpoint of $\widehat{BC}.$ It follows that $C = (\cos 20^{\circ}, \sin 20^{\circ}),$ so $t=20.$

  2. $BA=BC$

    Note that $B$ must be the midpoint of $\widehat{AC}.$ It follows that $C = (\cos 80^{\circ}, \sin 80^{\circ}),$ so $t=80.$

  3. $CA=CB$

    Note that $C$ must be the midpoint of $\widehat{AB}.$ It follows that $C = (\cos 50^{\circ}, \sin 50^{\circ})$ or $C = (\cos 230^{\circ}, \sin 230^{\circ}),$ so $t=50$ or $t=230.$

Together, the sum of all such possible values of $t$ is $20+80+50+230=\boxed{\textbf{(E)} \: 380}.$

Remark

The following diagram shows all possible locations of $C:$

[asy] /* Made by MRENTHUSIASM */ size(200);   int xMin = -1; int xMax = 1; int yMin = -1; int yMax = 1; int numRays = 36;  //Draws a polar grid that goes out to a number of circles  //equal to big, with numRays specifying the number of rays:  void polarGrid(int big, int numRays)  {   for (int i = 1; i < big+1; ++i)   {     draw(Circle((0,0),i), gray+linewidth(0.4));   }   for (int i=0;i<numRays;++i)    draw(rotate(i*360/numRays)*((-big,0)--(big,0)), gray+linewidth(0.4)); }  polarGrid(xMax,numRays); draw((xMin,0)--(xMax,0),black+linewidth(1.5),EndArrow(5)); draw((0,yMin)--(0,yMax),black+linewidth(1.5),EndArrow(5)); label("$x$",(xMax,0),2*E); label("$y$",(0,yMax),2*N);  pair A, B, C1, C2, C3, C4; A = dir(40); B = dir(60); C1 = dir(20); C2 = dir(80); C3 = dir(50); C4 = dir(230);  dot("$A$",A,1.5*dir(A),linewidth(4)); dot("$B$",B,1.5*dir(B),linewidth(4)); dot("$C_1$",C1,1.5*dir(C1),red+linewidth(4)); dot("$C_2$",C2,1.5*dir(C2),red+linewidth(4)); dot("$C_3$",C3,1.5*dir(C3),red+linewidth(4)); dot("$C_3$",C4,1.5*dir(C4),red+linewidth(4)); [/asy]

~Steven Chen (www.professorchenedu.com) ~Wilhelm Z ~MRENTHUSIASM

Video Solution by TheBeautyofMath

https://youtu.be/4qgYrCYG-qw?t=1304

~IceMatrix

See Also

2021 Fall AMC 12B (ProblemsAnswer KeyResources)
Preceded by
Problem 9
Followed by
Problem 11
1 2 3 4 5 6 7 8 9 10 11 12 13 14 15 16 17 18 19 20 21 22 23 24 25
All AMC 12 Problems and Solutions

The problems on this page are copyrighted by the Mathematical Association of America's American Mathematics Competitions. AMC logo.png